• Want to take part in this year's BoS Trials event for Maths and/or Business Studies?
    Click here for details and register now!
  • YOU can help the next generation of students in the community!
    Share your trial papers and notes on our Notes & Resources page

polynomials (1 Viewer)

Joined
Oct 23, 2005
Messages
116
Location
Fairfield West
Gender
Male
HSC
N/A
consider P(x) = ax^4 + bx^3 + cx^2 + dx + e, where a,b,c,d and e are integers.
Suppose w is an integer such that P(w) = 0.

(i) Prove that w divides e.

(ii) Prove that the polynomial Q(x) = 5x^4 + 2x^3 -3x^2 -x + 3 does not have an integer root.
 

Trebla

Administrator
Administrator
Joined
Feb 16, 2005
Messages
8,297
Gender
Male
HSC
2006
consider P(x) = ax^4 + bx^3 + cx^2 + dx + e, where a,b,c,d and e are integers.
Suppose w is an integer such that P(w) = 0.

(i) Prove that w divides e.

(ii) Prove that the polynomial Q(x) = 5x^4 + 2x^3 -3x^2 -x + 3 does not have an integer root.
(i)
P(w) = 0
=> aw4 + bw3 + cw2 + dw + e = 0
=> e = - w(aw3 + bw2 + cw + d)
Since a, b, c , d and w are integers then e = w x integer => w divides e

(ii)
Suppose Q(x) does have an integer root w then from (i)
3 = - w(5w3 + 2w2 - 3w - 1)
This means that for integer roots to exist w must be a factor of 3. If this is true then w is either 1 or 3 (the only factors of 3). However Q(1) and Q(3) are non-zero therefore we have a contradication so the assumption is not true hence we conclude there is no integer root.
 
Joined
Oct 23, 2005
Messages
116
Location
Fairfield West
Gender
Male
HSC
N/A
(i)
P(w) = 0
=> aw4 + bw3 + cw2 + dw + e = 0
=> e = - w(aw3 + bw2 + cw + d)
Since a, b, c , d and w are integers then e = w x integer => w divides e

(ii)
Suppose Q(x) does have an integer root w then from (i)
3 = - w(5w3 + 2w2 - 3w - 1)
This means that for integer roots to exist w must be a factor of 3. If this is true then w is either 1 or 3 (the only factors of 3). However Q(1) and Q(3) are non-zero therefore we have a contradication so the assumption is not true hence we conclude there is no integer root.
For part (ii), could w also possibly be -1 or -3 (since they're also factors of 3)?
 

Trebla

Administrator
Administrator
Joined
Feb 16, 2005
Messages
8,297
Gender
Male
HSC
2006
Yeah sorry, I forgot about them. Either way Q(-1) and Q(-3) should still be non-zero.
 

Users Who Are Viewing This Thread (Users: 0, Guests: 1)

Top